GMAT考试写作参考例文推荐

GMAT考试写作参考例文

GMAT考试写作参考例文推荐

The following appeared in a memo to the Saluda town council from the towns business manager.

Research indicates that those who exercise regularly are hospitalized less than half as often as those who dont exercise. By providing a well-equipped gym for Saludas municipal employees, we should be able to reduce the cost of our group health insurance coverage by approximately 50% and thereby achieve a balanced town budget.

Discuss how well reasoned . . . etc.

In this memo Saludas business manager recommends that the town provide a gym for its employees as a means of balancing the towns budget. The manager reasons that since studies show that people who exercise regularly are hospitalized less than half as often than those who dont exercise, Saluda could save approximately 50% on the cost of its group health insurance coverage by providing its employees with a well-equipped gym. The savings on insurance would balance the towns budget. The managers argument is unconvincing because it rests on several unsupported and dubious assumptions.

First, the manager assumes that Saludas employees will exercise regularly if a well-equipped facility is provided for them. This assumption is questionable since the mere fact that a gym is made available for employee use is no guarantee that they will avail themselves of it at all, let alone on a regular basis.